Show that for all integers congruent modulo 11

  • Thread starter Thread starter Lelouch
  • Start date Start date
  • Tags Tags
    Integers
Click For Summary
The discussion focuses on proving that for all integers n, the expression n^(11a + 21b + 31c) is congruent to n^(a + b + c) modulo 11. The approach involves demonstrating that 11 divides the difference between these two expressions, which can be shown using Fermat's Little Theorem. Participants suggest various methods, including breaking down variables and using induction, but ultimately confirm that applying Fermat's theorem simplifies the proof. The consensus is that the steps taken align with the theorem's principles, validating the approach. The discussion concludes with affirmation of the correctness of the method used.
Lelouch
Messages
18
Reaction score
0

Homework Statement


Let ##a, b, c \in \mathbb{N \setminus \{0 \}}##. Show that for all ##n \in \mathbb{Z}## we have

$$n^{11a + 21b + 31c} \equiv n^{a + b + c} \quad (mod \text{ } 11).$$

Homework Equations

The Attempt at a Solution



We have to show that ##11 | (n^{11a + 21b + 31c} - n^{a + b + c}) \iff \exists k \in \mathbb{Z} : n^{11a + 21b + 31c} - n^{a + b + c} = 11k.##

I tried showing that ##11 | n^{11a + 21b + 31c}## and ##11 | - n^{a + b + c}## and then conclude that ## 11 | (n^{11a + 21b + 31c} - n^{a + b + c})##. I also tried breaking down the variables in even and odd but that gave me too many cases which became tedious very fast. I also tried induction on a keeping b,c fixed.
 
Physics news on Phys.org
Have you tried to work with ##(n^{a+2b+3c})^{10} \equiv 0 \operatorname{mod} 11## and perhaps the divisibility criterion of ##11## (alternating digit sum)? Or consider which remainders modulo ##11## become ##0## if taken to the ##10-##th power.
 
What I've done is used Fermat's Little Theorem like this

\begin{equation*}
\begin{split}
n^{11a + 21b + 31c} & \equiv n^{11a}n^{21b}n^{31c} \text{ (mod 11)} \\
& \equiv (n^{a})^{11}(n^{b})^{11} (n^{b})^{10}(n^{c})^{11}(n^{c})^{11}(n^{c})^{9} \text{ (mod 11)} \\
& \equiv n^{a}n^{b} (n^{b})^{10}n^{c}n^{c}(n^{c})^{9} \text{ (mod 11)} \\
& \equiv n^{a}(n^{b})^{11}(n^{c})^{11} \text{ (mod 11)} \\
& \equiv n^{a}n^{b}n^{c} \text{ (mod 11)}.
\end{split}
\end{equation*}

Since 11 is a prime number we have by Fermat's Little Theorem that

\begin{equation*}
(n^{a})^{11} \equiv n^{a} \text{ (mod 11)} \quad \land \quad
(n^{b})^{11} \equiv n^{a} \text{ (mod 11)} \quad \land \quad (n^{c})^{11} \equiv n^{a} \text{ (mod 11)}.
\end{equation*}

Is this correct?
 
Yes, that's fine.
 
Question: A clock's minute hand has length 4 and its hour hand has length 3. What is the distance between the tips at the moment when it is increasing most rapidly?(Putnam Exam Question) Answer: Making assumption that both the hands moves at constant angular velocities, the answer is ## \sqrt{7} .## But don't you think this assumption is somewhat doubtful and wrong?

Similar threads

  • · Replies 10 ·
Replies
10
Views
2K
  • · Replies 2 ·
Replies
2
Views
1K
  • · Replies 1 ·
Replies
1
Views
2K
  • · Replies 1 ·
Replies
1
Views
3K
  • · Replies 3 ·
Replies
3
Views
2K
  • · Replies 3 ·
Replies
3
Views
2K
  • · Replies 3 ·
Replies
3
Views
1K
  • · Replies 11 ·
Replies
11
Views
2K
  • · Replies 3 ·
Replies
3
Views
2K
  • · Replies 5 ·
Replies
5
Views
1K